levaska savings rate

This topic has expert replies
Senior | Next Rank: 100 Posts
Posts: 80
Joined: Thu Aug 12, 2010 11:47 pm
Location: chennai, india
Thanked: 2 times

levaska savings rate

by kushal.adhia » Mon Sep 20, 2010 10:40 am
Five years ago, as part of a plan to encourage citizens of Levaska to increase the amount of money they put into savings, Levaska's government introduced special savings accounts in which up to $3,000 a year can be saved with no tax due on the interest unless money is withdrawn before the account holder reaches the age of sixty-five. Millions of dollars have accumulated in the special accounts, so the government's plan is obviously working.
Which of the following, if true, most seriously weakens the argument?

A. A substantial number of Levaskans have withdrawn at least some of the money they had invested in the special accounts.
B. Workers in Levaska who already save money in long-term tax-free accounts that are offered through their workplace cannot take advantage of the special savings accounts introduced by the government.
C. The rate at which interest earned on money deposited in regular savings accounts is taxed depends on the income bracket of the account holder.
D. Many Levaskans who already had long-term savings have steadily been transferring those savings into the special accounts.
E. Many of the economists who now claim that the government's plan has been successful criticized it when it was introduced.


I'm curious to know why the answer is not D.
OA is C



EXTREMELY EXTREMELY SORRY. THIS IS NOT THE QUESTION I HAD A PROBLEM WITH
I've re-posted the correct one... Sorry again
Last edited by kushal.adhia on Mon Sep 20, 2010 11:43 am, edited 1 time in total.

User avatar
Legendary Member
Posts: 866
Joined: Mon Aug 02, 2010 6:46 pm
Location: Gwalior, India
Thanked: 31 times

by goyalsau » Mon Sep 20, 2010 11:30 am
kushal.adhia wrote:Five years ago, as part of a plan to encourage citizens of Levaska to increase the amount of money they put into savings, Levaska's government introduced special savings accounts in which up to $3,000 a year can be saved with no tax due on the interest unless money is withdrawn before the account holder reaches the age of sixty-five. Millions of dollars have accumulated in the special accounts, so the government's plan is obviously working.
Which of the following, if true, most seriously weakens the argument?

A. A substantial number of Levaskans have withdrawn at least some of the money they had invested in the special accounts.
B. Workers in Levaska who already save money in long-term tax-free accounts that are offered through their workplace cannot take advantage of the special savings accounts introduced by the government.
C. The rate at which interest earned on money deposited in regular savings accounts is taxed depends on the income bracket of the account holder.
D. Many Levaskans who already had long-term savings have steadily been transferring those savings into the special accounts.
E. Many of the economists who now claim that the government's plan has been successful criticized it when it was introduced.


I'm curious to know why the answer is not D.
OA is C

I put in more than 5 minutes in this question and in the end marked D,
And when i saw C as a official answer, MY reaction Was OH MY GOD not again.
:cry: :cry: :cry: :cry: :cry: :cry: :cry: :cry: :cry: :cry:

option D
Many Levaskans who already had long-term savings have steadily been transferring those savings into the special accounts. I think MANY is the problem over here

Definition of many is
"Many" is an unspecified % and could just mean 1.

So if only one person is moving the saving then that does not make a big impact than the tax income as stated in option C.


https://www.beatthegmat.com/is-most-many ... t8823.html

Legendary Member
Posts: 1119
Joined: Fri May 07, 2010 8:50 am
Thanked: 29 times
Followed by:3 members

by diebeatsthegmat » Mon Sep 20, 2010 11:39 am
goyalsau wrote:
kushal.adhia wrote:Five years ago, as part of a plan to encourage citizens of Levaska to increase the amount of money they put into savings, Levaska's government introduced special savings accounts in which up to $3,000 a year can be saved with no tax due on the interest unless money is withdrawn before the account holder reaches the age of sixty-five. Millions of dollars have accumulated in the special accounts, so the government's plan is obviously working.
Which of the following, if true, most seriously weakens the argument?

A. A substantial number of Levaskans have withdrawn at least some of the money they had invested in the special accounts.
B. Workers in Levaska who already save money in long-term tax-free accounts that are offered through their workplace cannot take advantage of the special savings accounts introduced by the government.
C. The rate at which interest earned on money deposited in regular savings accounts is taxed depends on the income bracket of the account holder.
D. Many Levaskans who already had long-term savings have steadily been transferring those savings into the special accounts.
E. Many of the economists who now claim that the government's plan has been successful criticized it when it was introduced.


I'm curious to know why the answer is not D.
OA is C

I put in more than 5 minutes in this question and in the end marked D,
And when i saw C as a official answer, MY reaction Was OH MY GOD not again.
:cry: :cry: :cry: :cry: :cry: :cry: :cry: :cry: :cry: :cry:

option D
Many Levaskans who already had long-term savings have steadily been transferring those savings into the special accounts. I think MANY is the problem over here

Definition of many is
"Many" is an unspecified % and could just mean 1.

So if only one person is moving the saving then that does not make a big impact than the tax income as stated in option C.


https://www.beatthegmat.com/is-most-many ... t8823.html
the answer is indeed D, not C, recheck the OA

User avatar
MBA Student
Posts: 113
Joined: Sat Dec 05, 2009 10:05 pm
Location: West Lafayette
Thanked: 1 times
GMAT Score:700

by g000fy » Mon Sep 20, 2010 1:18 pm
I picked D. I googled to confirm the OA. Found D on another board.

User avatar
Legendary Member
Posts: 866
Joined: Mon Aug 02, 2010 6:46 pm
Location: Gwalior, India
Thanked: 31 times

by goyalsau » Tue Sep 21, 2010 2:55 am
kushal.adhia wrote:Five years ago, as part of a plan to encourage citizens of Levaska to increase the amount of money they put into savings, Levaska's government introduced special savings accounts in which up to $3,000 a year can be saved with no tax due on the interest unless money is withdrawn before the account holder reaches the age of sixty-five. Millions of dollars have accumulated in the special accounts, so the government's plan is obviously working.
Which of the following, if true, most seriously weakens the argument?

A. A substantial number of Levaskans have withdrawn at least some of the money they had invested in the special accounts.
B. Workers in Levaska who already save money in long-term tax-free accounts that are offered through their workplace cannot take advantage of the special savings accounts introduced by the government.
C. The rate at which interest earned on money deposited in regular savings accounts is taxed depends on the income bracket of the account holder.
D. Many Levaskans who already had long-term savings have steadily been transferring those savings into the special accounts.
E. Many of the economists who now claim that the government's plan has been successful criticized it when it was introduced.


I'm curious to know why the answer is not D.
OA is C



EXTREMELY EXTREMELY SORRY. THIS IS NOT THE QUESTION I HAD A PROBLEM WITH
I've re-posted the correct one... Sorry again
What's the correct Answer of this question?
C or D.

g000fy said that official answer is D.
So what's the correct Answer?

Legendary Member
Posts: 2326
Joined: Mon Jul 28, 2008 3:54 am
Thanked: 173 times
Followed by:2 members
GMAT Score:710